LSAT and Law School Admissions Forum

Get expert LSAT preparation and law school admissions advice from PowerScore Test Preparation.

User avatar
 Dave Killoran
PowerScore Staff
  • PowerScore Staff
  • Posts: 5852
  • Joined: Mar 25, 2011
|
#40818
Complete Question Explanation
(The complete setup for this game can be found here: lsat/viewtopic.php?t=15545)

The correct answer choice is (C)

If P is assigned to the middle floor with exactly two other departments (one of which must be T from the first rule, the other of which must be H or I), then the 3-3-1 distribution is in effect. Depending on the placement of L, either template is still possible. Drawing out these templates might seem like the best approach, but the distribution alone is powerful enough to answer this question.

In the 3-3-1, the following assignment of variables must occur:
PT59_Game_#1_setup_diagram 8.png
Accordingly, F and S are assigned to the same floor, and answer choice (C) must be true, and is therefore correct.
You do not have the required permissions to view the files attached to this post.

Get the most out of your LSAT Prep Plus subscription.

Analyze and track your performance with our Testing and Analytics Package.